LSAT and Law School Admissions Forum

Get expert LSAT preparation and law school admissions advice from PowerScore Test Preparation.

 Administrator
PowerScore Staff
  • PowerScore Staff
  • Posts: 8916
  • Joined: Feb 02, 2011
|
#59615
Complete Question Explanation
(The complete setup for this game can be found here: lsat/viewtopic.php?t=26510)

The correct answer choice is (E)

Within the templates, the only time J can be the fifth group is under Template #2:

pt43_j04_g1_q4a.png
If J is in group 5, them M must be in group 6:

pt43_j04_g1_q4b.png
The only remaining uncertainty is the placement of G and P, which can rotate between the first and third groups:

pt43_j04_g1_q4c.png
Because this is Must be true question, you can use the hypothetical above to quickly accelerate through the questions. In this instance, answer choice (E) is quickly proven correct.

Answer choice (A): Although P could be in the first group, P could also be in the third group, and so this answer choice does not have to be true. Note how the test makers immediately attack you on the uncertainty within this question.

Answer choice (B): F must be in the second group, not the first group.

Answer choice (C): V must be in the fourth group, not the second group.

Answer choice (D): Although G could be in the third group, G could also be in the first group, and so this answer choice does not have to be true.

Answer choice (E): This is the correct answer choice.
You do not have the required permissions to view the files attached to this post.

Get the most out of your LSAT Prep Plus subscription.

Analyze and track your performance with our Testing and Analytics Package.